Mathcenter Forum  

Go Back   Mathcenter Forum > คณิตศาสตร์มัธยมศึกษา > ปัญหาคณิตศาสตร์ ม. ต้น
สมัครสมาชิก คู่มือการใช้ รายชื่อสมาชิก ปฏิทิน ข้อความวันนี้

ตั้งหัวข้อใหม่ Reply
 
เครื่องมือของหัวข้อ ค้นหาในหัวข้อนี้
  #1  
Old 01 ตุลาคม 2008, 08:36
Dr.K's Avatar
Dr.K Dr.K ไม่อยู่ในระบบ
ลมปราณบริสุทธิ์
 
วันที่สมัครสมาชิก: 04 พฤษภาคม 2007
ข้อความ: 100
Dr.K is on a distinguished road
Default เลข ยก กำ ลัง ครับ (การเปรียบเทียบกัน)

มีแนวคิด อย่างไร ครับ
เช่น 3 ยกกำลัง7 เทียบกับ 7 ยกกำลัง 3 อะไร มากกว่ากัน
a ยกกำลัง b เทียบกับ b ยกกำลัง a อะไร มากกว่ากัน
__________________
I love Badminton!
ตอบพร้อมอ้างอิงข้อความนี้
  #2  
Old 01 ตุลาคม 2008, 10:30
Julian's Avatar
Julian Julian ไม่อยู่ในระบบ
บัณฑิตฟ้า
 
วันที่สมัครสมาชิก: 18 กันยายน 2008
ข้อความ: 348
Julian is on a distinguished road
Default

ถ้าเราจะเทียบเราก็เทียบให้เลขชี้กำลังมันเท่ากัน ทำแบบนี้ครับ

$3^7$ = $( 3 x 3 x 3 x 3 )^3$

= $81^3$

แล้วเราก็เปรียบเทียบ $81^3 กับ 7^3 $

แล้ว ก็ได้คำตอบล่ะคับ
ตอบพร้อมอ้างอิงข้อความนี้
  #3  
Old 01 ตุลาคม 2008, 11:04
หยินหยาง's Avatar
หยินหยาง หยินหยาง ไม่อยู่ในระบบ
กระบี่จักรวาล
 
วันที่สมัครสมาชิก: 06 มกราคม 2007
ข้อความ: 2,921
หยินหยาง is on a distinguished road
Default

อ้างอิง:
ข้อความเดิมเขียนโดยคุณ Julian View Post
ถ้าเราจะเทียบเราก็เทียบให้เลขชี้กำลังมันเท่ากัน ทำแบบนี้ครับ

$3^7$ = $( 3 x 3 x 3 x 3 )^3$

= $81^3$

แล้วเราก็เปรียบเทียบ $81^3 กับ 7^3 $

แล้ว ก็ได้คำตอบล่ะคับ
$3^7 \not=81^3$ เพราะว่า $81^3=3^{12}$ครับ
ตอบพร้อมอ้างอิงข้อความนี้
  #4  
Old 01 ตุลาคม 2008, 11:35
กรza_ba_yo's Avatar
กรza_ba_yo กรza_ba_yo ไม่อยู่ในระบบ
กระบี่ประสานใจ
 
วันที่สมัครสมาชิก: 06 ธันวาคม 2007
ข้อความ: 772
กรza_ba_yo is on a distinguished road
Default

เเล้วถ้าเกิดมันเกิดเลขยกกำลังมันไม่เท่ากันละคับ
__________________
คนเราหากล้มก็ต้องลุก
ผู้ใดล้มเเล้วไม่ลุกผู้นั้นยิ่งกว่าสุนัข
สุนัขมันล้มเเล้วมันยังลุกได้
เเล้วทำไมคนถึงจะลุกไม่ได้
ตอบพร้อมอ้างอิงข้อความนี้
  #5  
Old 01 ตุลาคม 2008, 12:47
Julian's Avatar
Julian Julian ไม่อยู่ในระบบ
บัณฑิตฟ้า
 
วันที่สมัครสมาชิก: 18 กันยายน 2008
ข้อความ: 348
Julian is on a distinguished road
Default

ขอบคุณครับ ผมเองรู้สึกมึนๆกับเรื่องนี้เหมือนกัน

งั้นก็ต้องใช้วิธีนี้

ทำให้เลขชี้กำลังมันเท่ากันเหมือนเดิมแต่ดูที่ตัวเลขคือแบบนี้คับ

$3^{(\frac{7}{3} )3}$ คือมันน่าจะมากกว่า 9 ไปเล็กน้อยแล้วยกกำลัง 3

แล้วเทียบกับ 7 ยกกำลัง 3

จะได้ว่า $3^7 > 7^3$
ตอบพร้อมอ้างอิงข้อความนี้
  #6  
Old 01 ตุลาคม 2008, 14:00
Puriwatt's Avatar
Puriwatt Puriwatt ไม่อยู่ในระบบ
ลมปราณไร้สภาพ
 
วันที่สมัครสมาชิก: 14 กันยายน 2006
ข้อความ: 1,435
Puriwatt is on a distinguished road
Default

ลองใช้วิธีหารด้วย $3^3$ ดูซิครับ แล้วเราจะได้การเปรียบเทียบใหม่คือ $\frac{3^7}{3^3} = 3^4$ กับ $ (\frac {7}{3})^3= (2\frac {1}{3})^3$

เราจะพบว่า $3^4$ > $(2\frac {1}{3})^3$ มากกว่ากันอย่างชัดเจนครับ
ตอบพร้อมอ้างอิงข้อความนี้
  #7  
Old 01 ตุลาคม 2008, 18:24
[SIL]'s Avatar
[SIL] [SIL] ไม่อยู่ในระบบ
กระบี่ไร้สภาพ
 
วันที่สมัครสมาชิก: 12 กรกฎาคม 2008
ข้อความ: 1,520
[SIL] is on a distinguished road
Default

วิธีพี่ Puriwat เข้าใจง่ายสุดแล้วครับความจริง $3^7 , 7^3$ อ่ะมันไม่ยากเท่าไหร่
แต่ถ้าเป็น $\sqrt{2} ,\sqrt[3]{3} ,\sqrt[6]{6}$ นี่มีแนวคิดยังไงหรอครับ(พอดีเคยเป็นข้อสอบ สอวน.) ถ้าไปทำจริงกรรมการคงไม่ให้เอาตารางล็อกเข้าไปหรอกครับถึงจะได้ตารางล็อคมันก็ไม่ค่อยแม่นอยู่ดีอ่ะ
ตอบพร้อมอ้างอิงข้อความนี้
  #8  
Old 01 ตุลาคม 2008, 18:36
MirRor's Avatar
MirRor MirRor ไม่อยู่ในระบบ
บัณฑิตฟ้า
 
วันที่สมัครสมาชิก: 03 มีนาคม 2008
ข้อความ: 394
MirRor is on a distinguished road
Default

อ้างอิง:
ข้อความเดิมเขียนโดยคุณ [SIL] View Post

แต่ถ้าเป็น $\sqrt{2} ,\sqrt[3]{3} ,\sqrt[6]{6}$


ยกกำลัง 6 ไปดื้อๆ เลย ทำให้ ไม่ติดรูทอ่ะ
__________________
ตอบพร้อมอ้างอิงข้อความนี้
  #9  
Old 01 ตุลาคม 2008, 18:39
[SIL]'s Avatar
[SIL] [SIL] ไม่อยู่ในระบบ
กระบี่ไร้สภาพ
 
วันที่สมัครสมาชิก: 12 กรกฎาคม 2008
ข้อความ: 1,520
[SIL] is on a distinguished road
Default

เห้ยจริงด้วย ก๊ากๆๆๆ สมเพชตัวเองอารายว้าเนี่ยคุณ MirRor ขอบคุณครับ
หมดกันแล้วเรารีบปิดกระทู้ด่วนครับอย่าให้ใครผ่านมาเห็น TT
ตอบพร้อมอ้างอิงข้อความนี้
  #10  
Old 01 ตุลาคม 2008, 18:43
Julian's Avatar
Julian Julian ไม่อยู่ในระบบ
บัณฑิตฟ้า
 
วันที่สมัครสมาชิก: 18 กันยายน 2008
ข้อความ: 348
Julian is on a distinguished road
Default

ถ้าจะเปรียบเทียบอย่างนั้นก็ต้องทำเลขชี้กำลังให้เท่ากันครับ

ขั้นตอนแรก หา หรม ของ $\frac{1}{2},\frac{1}{3},\frac{1}{6} ได้ \frac{1}{6} ครับ $

เราต้องจัดให้มันอยู่ในรูป $A^\frac{1}{6} $ ก่อนแล้วเปรียบเทียบกัน

$\sqrt{2} = 2^{\frac{1}{2} }$ ถูกไหมครับ ทำให้มันอยู่ในรูป $A^\frac{1}{6} $

คือ $[2^3]^\frac{1}{6} $

แล้วเราก็ทำเช่นนี้กับ $\sqrt[3]{3} และ \sqrt[6]{6} $

ได้ออกมาคือ $ [3^2]^{\frac{1}{6} } และ [6]^{\frac{1}{6} }$

แล้วเราก็เปรียบเทียบทั้งสามจำนวน

จะได้ว่า จำนวนที่มากที่สุด คือ $[3^2]^{\frac{1}{6} } = \sqrt[3]{3} $

รองลงมา คือ $[2^3]^\frac{1}{6} = \sqrt{2}$

และที่น้อยที่สุด คือ $[6]^{\frac{1}{6} = \sqrt[6]{6} }$
ตอบพร้อมอ้างอิงข้อความนี้
ตั้งหัวข้อใหม่ Reply



กฎการส่งข้อความ
คุณ ไม่สามารถ ตั้งหัวข้อใหม่ได้
คุณ ไม่สามารถ ตอบหัวข้อได้
คุณ ไม่สามารถ แนบไฟล์และเอกสารได้
คุณ ไม่สามารถ แก้ไขข้อความของคุณเองได้

vB code is On
Smilies are On
[IMG] code is On
HTML code is Off
ทางลัดสู่ห้อง


เวลาที่แสดงทั้งหมด เป็นเวลาที่ประเทศไทย (GMT +7) ขณะนี้เป็นเวลา 05:17


Powered by vBulletin® Copyright ©2000 - 2024, Jelsoft Enterprises Ltd.
Modified by Jetsada Karnpracha